HESI: MAT, MENTAL HEALTH, N PEDS

अब Quizwiz के साथ अपने होमवर्क और परीक्षाओं को एस करें!

Which component of the human personality, according to Freud, allows an individual to judge reality accurately? Multiple choice question Id Ego Superego Oedipus complex

According to Freud the ego allows an individual to judge reality accurately. The id is basic instinctual impulses driven to achieve pleasure. The superego is often referred to as the conscience. The Oedipus complex is a belief that a female child fantasizes about her father as a love interest.

At what age should a nurse instruct a parent that his or her child will have finicky eating habits? Multiple choice question One year old Two years old Four years old Five years old

Four-year-old children have finicky eating habits. Toddlers (one- and two-year-old children) often develop the desire to eat one food repeatedly. Five-year-olds are more interested in trying new foods.

Which sources of stress should the nurse plan to include in the assessment for a 9-year-old client who is scheduled for a routine health maintenance visit? Multiple selection question Stature Fair play Propriety Interruptions Rebelliousness

Common sources of stress for a 9-year-old client include fair play, propriety, interruptions, and rebelliousness. Stature becomes a source of stress for the school-age child who is 10 to 12 years of age.

A newborn male is circumcised. What is the most essential nursing assessment during the initial postoperative period? Multiple choice question Bleeding Infection Shrill, piercing cry Decreased urine output

bleeding The penis is a vascular area, and the infant must be monitored closely for bleeding. It is too soon to detect signs of infection. Although a circumcised infant may be uncomfortable, he can be medicated for pain; this type of cry may be indicative of central nervous system damage. Decreased urine output is usually not a problem with circumcision.

On the third day of a 2-year-old toddler's hospitalization the nurse notes that the child, who had been screaming and crying inconsolably, has begun to regress and is now lying quietly in the crib with a blanket. What stage of separation anxiety has developed? Multiple choice question Denial Despair Mistrust Rejection

despair The second stage of separation anxiety is despair, in which the child is depressed, lonely, and uninterested in the surroundings. The third stage of separation, denial or detachment, occurs later as hospitalization becomes prolonged. The child is suffering from separation anxiety, which does not include a stage of mistrust or of rejection.

A client had a fourth-degree perineal laceration during the birth of her neonate. What should the nurse recommend to protect the area from additional trauma? Multiple choice question "Take sitz baths at least three times each day." "Apply a premoistened anesthetic pad to the area." "Avoid straining at stool with the use of an enema." "Eat a high-fiber diet with increased fluid intake."

"Eat a high-fiber diet with increased fluid intake." Fluid intake and fiber help promote soft stools and defecation. Promotion of defecation is a priority because a fourth-degree laceration impinges on the rectal sphincter. Constipation will further traumatize the rectum. Although sitz baths and anesthetic pads each relieve pain and promote healing, they do not prevent additional trauma. An enema would cause additional trauma to the rectum and is contraindicated.

Which treatment should the nurse prepare to administer when providing care to a toddler-age client who presents after an accidental overdose of aspirin? Multiple choice question Gastric lavage Activated charcoal Peritoneal dialysis Vitamin D injection

Activated charcoal The nurse would prepare to administer activated charcoal and repeat every 4 hours, if needed, for a client with active bowel sounds. Gastric lavage will not remove concentrations of aspirin. Hemodialysis, not peritoneal dialysis, is a treatment that may be prescribed for a client who presents with an overdose of aspirin. Vitamin K, not D, is administered to assist with clotting.

An 18-year-old adolescent who was diagnosed with new-onset type 1 diabetes mellitus has stress and reports not having a menstrual cycle for a long time. Which condition is the adolescent experiencing? Multiple choice question Amenorrhea Primary amenorrhea Female athlete triad Hypogonadotropic amenorrhea

Hypogonadotropic amenorrhea may occur in type 1 diabetic adolescents experiencing stress. This condition can also result from sudden and severe weight loss, eating disorders, strenuous exercise, and mental illness.

What situation would the nurse instruct the parents may possibly predispose a preschooler to feeling guilty? Multiple choice question The caregiver's responses are too harsh. The individual is unable to establish companionship. The parents try to control the child and limit choices. The parents fail to establish a sense of trust in the child.

The caregiver's responses are too harsh. Preschoolers often develop a feeling of guilt if the caregiver's responses are too harsh. If a young adult is not able to establish companionship and intimacy, isolation results due to rejection and disappointment. The toddler develops his or her autonomy by making choices. Controlling the child and limiting choices can predispose a child to a sense of shame and doubt. If parents fail to establish a sense of trust with an infant, it can lead to a feeling of mistrust in the infant.

According to Erikson, a person's adjustment to the period of senescence will depend largely on the adjustment the individual made to which earlier developmental stage? Multiple choice question Trust versus mistrust Industry versus inferiority Identity versus role confusion Generativity versus stagnation

Generativity versus stagnation Erikson theorized that how well people adapt to the current stage depends on how well they adapted to the stage immediately preceding it—in this instance, adulthood. Although Erikson believed that the strengths and weaknesses of each stage are present in some form in all succeeding stages, their influence decreases with time.

An infant is found to have communicating hydrocephalus. The parents ask for clarification of the primary healthcare provider's explanation of the problem. How should the nurse respond? Multiple choice question "Too much spinal fluid is being produced within the spaces (ventricles) of the brain." "The flow of spinal fluid through the brain cells does not empty effectively into the spinal cord." "The spinal fluid is prevented from being adequately absorbed by a blockage in the spaces (ventricles) of the brain." "There is a part of the brain surface that usually absorbs spinal fluid after its production that is not functioning adequately."

"There is a part of the brain surface that usually absorbs spinal fluid after its production that is not functioning adequately." In communicating hydrocephalus, a part of the brain surface that usually absorbs spinal fluid after its production does not function adequately. Too much spinal fluid is often a result of a choroid plexus tumor. It does not interfere with the flow of cerebrospinal fluid through the ventricles. Stating that the flow of spinal fluid through the brain cells does not empty effectively into the spinal cord is inaccurate; brain cells and the spinal cord are not involved. Stating that the spinal fluid is prevented from being adequately absorbed by a blockage in the spaces (ventricles) of the brain reflects the pathophysiologic process of noncommunicating hydrocephalus.

A 49-year-old client is admitted with a diagnosis of cervical cancer. As the nurse is obtaining her health history, she says, "I haven't had a Pap smear for more than 5 years. I probably wouldn't be in the hospital today if I'd had those tests more often." What is the nurse's most appropriate response? Multiple choice question "Please tell me why you waited so long." "You feel as though you've neglected your health." "It's never too late to start taking care of yourself." "Most women hate to have Pap smears done, but they're really important."

"You feel as though you've neglected your health." Stating that the client feels that she's neglected her health indicates recognition of expressed feelings; a nondirective and reflective response encourages verbalization. Asking the client why she waited so long ignores the client's current emotional needs; direct statements often do not elicit feelings and may cut off communication. Stating that it is never too late to start taking care of her health is a judgmental response, because it implies that the client has been negligent. Although it is true that most clients hate to have Pap smears, this statement ignores the client's current emotional needs.

A client who has had a lumpectomy of the breast is about to undergo radiation therapy. What should the nurse's initial action be when the client visits the surgeon's office for the first postoperative appointment? Multiple choice question Provide a protective skin lotion. Assess the extent of wound healing. Teach sterile technique for skin care. Demonstrate how to dispose of urine safely.

Assess the extent of wound healing. Radiation will interfere with wound healing if it is initiated too soon; inadequate healing should be reported to the primary healthcare provider. Topical preparations should not be used unless prescribed. Sterile technique is not necessary unless there is a break in the skin. Urine and other excreta of a client receiving radiation to the breast area are not affected by the radiation.

The nurse at a prenatal clinic determines the fundal height of a healthy multipara at 16 weeks' gestation to be one fingerbreadth above the umbilicus. What might the nurse's next action be? Multiple choice question Check for two distinct fetal heart rates Ascertain the birth weights of the client's other children Inform the client that she may be mistaken about her due date Instruct the client about appropriate weight gain during pregnancy

Check for two distinct fetal heart rates Twins should be suspected with a faster-than-expected increase in fundal height; the nurse should assess the client for two distinct heartbeats. Fundal height, not the size of the fetus, should prompt the nurse to suspect a multiple pregnancy. The due date cannot be determined until ultrasonography has been performed. Weight gain does not influence the height of the fundus.

A nurse is caring for a client with a somatoform disorder. What should the nurse anticipate that this client will do? Multiple choice question Write down conversations to facilitate the recall of information. Monopolize conversations about the anxiety being experienced. Redirect the conversation with the nurse to physical symptoms. Start a conversation asking the nurse to recommend palliative care.

Clients with somatoform disorders are preoccupied with the symptoms that are being experienced and usually do not want to talk about their emotions or relate them to their current situation. Clients with somatoform disorders do not seek opportunities to discuss their feelings. Memory problems are not associated with somatoform disorders. These clients want and seek treatment, not palliative care.

The nurse is counseling the parents of an adolescent child on the benefits of social development in the child's maturation process. Which statement by the nurse needs correction to convey an appropriate message to the parents? Multiple choice question Feelings of immortality in the child are undesirable and should be condemned. The family should encourage the child to make relationships outside the family. Feelings of intense sociability and equally intense loneliness are normal in the child. The family should encourage the child to develop an identity independent of parental authority.

Feelings of immortality in the child are undesirable and should be condemned. Social development is a critical aspect of a child's maturation process. The feelings of immortality and release from the results of risky behavior are common in adolescent children. Although such feelings can be dangerous, they function as important developmental tools. These feelings help the child gain courage to build a separate self-identity by freeing himself or herself from family domination. Therefore the nurse should explain the significance of these feelings even though they seem negative. The nurse should instruct the parents to encourage the child to form social relationships outside the family. This helps in emancipating the child. The nurse should explain to the parents that adolescence is a transition period during which a child experiences various emotions ranging from intense sociability to intense loneliness. The parents should support and encourage the child to develop an independent identity apart from the family authority to achieve full maturity.

The nurse suggests counseling for a 13-year-old whose close friend has just committed suicide. The nurse's intervention is based on the understanding that an adolescent is at risk for copycat suicide mainly because members of this age group exhibit which characteristic? Multiple choice question Generally have poor impulse control Have had few experiences with mortality Often forge very close peer relationships Typically mimic the behavior of their peers

Generally have poor impulse control Adolescents are at especially high risk because of the immaturity of the prefrontal cortex. This is the portion of the brain that is responsible for judgment and impulse control. Although lack of life experience, the closeness of peer relationships, and behavioral mimicking are all characteristics of this age group, they are not as influential in a behavior such as copycat suicide.

Which interventions does the nurse implement to empower a family who has a child with Down syndrome? Multiple selection question Ask the family to engage in spiritual activities. Help the family recognize the possible stressors. Encourage the use of problem-solving strategies. Encourage more out-of-home activities for the parents. Refer the family to support groups and Internet resources.

Help the family recognize the possible stressors. Encourage the use of problem-solving strategies. Refer the family to support groups and Internet resources. The nurse understands that the family experiences multiple stressors and helps the family recognize those stressors. The nurse encourages the family to use effective problem-solving skills that convey support and care and have a calming influence on the child. The nurse also identifies proper support groups for the family to relieve stress. The use of Internet resources will help the family understand more about the child's disorder. Asking the family to engage in spiritual activities is not appropriate, because spirituality is a personal lifestyle choice. The nurse encourages the parents to spend more time at home to provide care for the child, as opposed to engaging in more out-of-home activities.

Fetal heart rate tracing abnormalities are observed on the fetal monitor when a client in active labor turns to the supine position. Which nursing action is most beneficial at this time? Multiple choice question Helping the client change her position Informing the client of the problem with the fetus Administering oxygen by mask to the client at 2 L/min Readjusting placement of the fetal monitor on the client's abdomen

Helping the client change her position Changing the maternal position is the most beneficial action, especially with late- and variable-deceleration patterns, because this position change will increase placental perfusion. Although the client should be kept informed of the fetus's condition, this may be done during or immediately after the position change; the needs of the fetus are the priority. If oxygen is used, the concentration should be greater than 2 L/min. Readjusting placement of the fetal monitor may be done after the position change; the immediate needs of the fetus are the priority.

A 65-year-old man is admitted to a mental health facility with a diagnosis of substance-induced persisting dementia resulting from chronic alcoholism. When conducting the admitting interview, the nurse determines that the client is using confabulation. What does the nurse recall precipitates the client's use of confabulation? Multiple choice question Ideas of grandeur Need for attention Marked memory loss Difficulty in accepting the diagnosis

Marked memory loss A client with this disorder has a loss of memory and adapts by filling in areas that cannot be remembered with made-up information. Ideas of grandeur do not occur with this type of dementia. The use of confabulation is not attention-seeking behavior; the individual is attempting to mask memory loss. This person is not coping with the diagnosis; when confabulating, the individual is attempting to mask memory loss.

A client visits a primary healthcare provider because of painful urination. The primary healthcare provider confirms that the client has candidiasis. Which medications would be prescribed? Multiple selection question Tinidazole Miconazole Clotrimazole Azithromycin Metronidazole

Miconazole Clotrimazole Miconazole and clotrimazole are used to treat candidiasis. Tinidazole is used to treat trichomoniasis. Azithromycin is used to treat chlamydia. Metronidazole is used to treat bacterial vaginosis.

A client with recurrent episodes of depression comes to the mental health clinic for a routine follow-up visit. The nurse suspects that the client is at increased risk for suicide. What is a contributing factor to the client's risk for suicide? Multiple choice question Psychomotor retardation Decreased physical activity Deliberate thoughtful behavior Overwhelming feelings of guilt

Overwhelming feelings of guilt contribute to the client's risk for suicide. The client may ruminate over past or current failings, and extreme guilt can assume psychotic proportions. Psychomotor retardation and decreased physical activity are clinical findings associated with depression and usually do not lead to suicide because the client does not have the energy for self-harm. Impulsive behaviors, not deliberate thoughtful behaviors, contribute to the client's risk for suicide.

A client is admitted with the diagnosis of borderline personality disorder and possible depression. The client has a history of abusive acting-out behavior. What is most important to assess when caring for this client? Multiple choice question Degree of anger Potential for suicide Level of intelligence Ability to test reality

Potential for suicide Depressed clients may use suicide as the ultimate escape from feelings; ensuring safety by protecting the client from self-harm is the priority. Although degree of anger is important, it is not the priority. Assessment of the level of intelligence is unnecessary; clients with a diagnosis of borderline personality disorder are usually of average intelligence. Clients with a diagnosis of borderline personality disorder are more concerned with satisfying their needs than testing reality; they are more concerned about themselves than others or the environment.

What nursing care to prevent a crisis is the same for school-aged children with sickle cell anemia and celiac disease? Multiple choice question Limiting activity Protecting the child from infection Documenting the color and consistency of stools Offering a low-carbohydrate, high-protein, low-fat die

Protecting the child from infection Children with both illnesses have inadequate resistance to infection. Sickling results from a low oxygen level; celiac crisis results from malnourishment and immunologic defects. Activity need not be limited in celiac disease; strenuous activity should be limited in sickle cell anemia. Documenting the color and consistency of stools is important for children with celiac disease; it is not necessary for children with sickle cell anemia. A low-carbohydrate, high-protein, low-fat diet is not particularly helpful for children with sickle cell anemia or celiac disease

An older client with vascular dementia has difficulty following simple directions for selecting clothes to be worn for the day. What does the nurse identify as the cause of these problems? Multiple choice question Receptive aphasia Impaired judgment Decreased attention span Clouding of consciousness

Receptive aphasia[1][2] interferes with interpreting and defining words in addition to following directions and selecting clothes. Following directions does not require skill in judgment or decision making. The selection of clothes does not require an intact attention span. Dementia does not cause a clouding of consciousness; delirium does.

What should the nurse include when developing a plan of care for a client in the manic phase of bipolar disorder? Multiple choice question Focusing the client's interest in reality Encouraging the client to talk as much as needed Persuading the client to complete any task that has been started Redirecting the client's excess energy to more constructive activities

Redirecting the client's excess energy to more constructive activities The hyperactive client usually is easily distracted, so excess energy can be redirected into constructive channels. There is nothing to indicate that the client is not in touch with reality. The client will talk a great deal with no encouragement. The client will not be able to focus long enough on one task to finish it.

It is determined that a staff nurse has a drug abuse problem. What approach to the staff nurse's addiction should be taken as an initial intervention? Multiple choice question Counseled by the staff psychiatrist Dismissed from the job immediately Referred to the employee assistance program Forced to promise to abstain from drugs in the future

Referral to the employee assistance program is a nonpunitive approach that attempts to help the nurse as an individual and as a professional. Counseling by the staff psychiatrist may be necessary for long-term therapy but is not the initial approach. Dismissing the nurse from the job immediately is a punitive nontherapeutic response that offers no chance of rehabilitation. The client has an addiction problem; promises will not keep the client from abusing drugs.

A client's diagnosis reports indicate the presence of syphilis, but the clinical manifestations of syphilis are absent in the client. Which stage of syphilis is the client in? Multiple choice question Latent Tertiary Primary Secondary

Syphilis is a sexually transmitted disease caused by Treponema pallidum. A client in the latent stage of syphilis shows a positive specific treponemal antibody test, but clinical manifestations are absent. Clinical manifestations in the tertiary stage of syphilis are presence of the gummas on the skin. Clinical manifestations in the primary stage of syphilis are chancres and genital ulcers. The clinical manifestations in the secondary stage of syphilis are malaise, fever, sore throat, headaches, and fatigue.

The school nurse is conducting a teacher's in-service on signs that may indicate that a child is a victim of bullying. Which sign should the nurse include in the teaching session? Multiple choice question The child wants to try out for the basketball team. The child asks for extra work to make better grades. The child is participating in several extracurricular activities after school. The child asks to go to the nurse's office frequently with vague complaints.

The child asks to go to the nurse's office frequently with vague complaints. Signs that may indicate a child is being bullied are similar to signs of other types of stress, including nonspecific ailments or complaints. Spending inordinate amounts of time in the school nurse's office with vague complaints is a sign that should be included in the teaching session. Withdrawal and deteriorating school performance are often signs of bullying. The child's wanting to participate on the basketball team, asking for extra work, and participating in extracurricular activities are not signs of withdrawal or deterioration in school performance.

A school-age child is mostly at home after school and gets engrossed in video games. Once a week, the child engages in swimming activity. The child's family also often orders fast food from restaurants. What does the nurse need to inform the parents about the child? Multiple choice question "The child is at risk for dental caries." "The child needs more reading activity." "The child is at risk for childhood obesity." "Fast food is not good for sports activity."

The child has a sedentary lifestyle, except for the swimming activity, and consumes fast food often. Therefore the child is at an increased risk for childhood obesity. The child will be at risk for dental caries if there is excessive sweetened beverages in the diet. Asking the parents to engage the child in more reading activity may be good, but it is more important to advise the parents to increase the physical activity and limit fast food to prevent obesity. The importance of fast food for sports activity is not a concern at the moment. It is more important to make the parents aware of the effects of such a lifestyle.

A worried mother reports that her teenage child wears unusual clothes and make-up and refuses to wear the clothes the mother buys. What does the nurse suspect as the cause for this behavior? Multiple choice question The child is establishing self-image in relation to others. The child is establishing gender role identification with the mother. The child is displaying group identity to develop a personal identity. The child is identifying himself or herself as a separate entity from the mother.

The child is displaying group identity to develop a personal identity. The child is seeking to establish a group identity by dressing and wearing make-up in a way that confirms this identity to other members of the group. The child establishes a self-image in relation to others during the latter part of childhood. In early childhood, the child attempts to establish gender role identification with the same-sex parent. During infancy, the child identifies himself or herself as a separate entity from the mother.

After a cocaine binge an individual is found unconscious and is admitted to the hospital with acute cocaine toxicity. What should the initial nursing action be directed toward? Multiple choice question Being understanding Establishing a patent airway Maintaining a drug-free environment Establishing a therapeutic relationship

The client is unconscious and unable to meet physical needs; a patent airway, breathing, and circulation are essential needs. Understanding and support are important once the client's physical condition has stabilized. Maintaining a drug-free environment will be a priority later in the treatment program. Establishment of a therapeutic relationship will increase in importance once the client's physical condition has stabilized.

A client with a long history of alcohol abuse who has been hospitalized for 1 week tells the nurse, "I feel much better and probably won't need any more treatment." What does the nurse conclude when evaluating the client's progress? Multiple choice question The client has accepted the illness and now must use willpower to resist alcohol. The client will probably not use alcohol again as long as the client's family remains supportive. The client's lack of insight into the emotional aspects of the illness indicates the need for continued supervision. The client's statement should be communicated to the practitioner so aversion therapy can be started before the client's discharge.

The client's lack of insight into the emotional aspects of the illness indicates the need for continued supervision. The client's statement indicates denial. The basic problem that led to the alcoholism has not been resolved, and therefore continued supervision is required. It is not true that the client has accepted the illness and now must use willpower to resist alcohol; the client is still denying the illness, and willpower alone will not keep the client away from alcohol. It may be true that the client probably will not use alcohol again as long as the client's family remains supportive, but it does not ensure compliance or successful rehabilitation. Aversion therapy will not be helpful unless the client understands the basic problem and how to resolve it.

A client in the outpatient clinic is denying that he is addicted to alcohol. He tells the nurse that he is not an alcoholic and that it is his nagging wife who causes him to drink. What is the most therapeutic response by the nurse? Multiple choice question "I don't think that your wife is the problem." "Everyone is responsible for his own actions." "Perhaps you should have marriage counseling." "Why do you think that your wife is the cause of your problems?"

The comment "Everyone is responsible for his own actions" encourages the client to accept responsibility and does not support denial as a defense mechanism. Although the comment "I don't think that your wife is the problem" may be true, it may also close off communication; with a decrease in communication the nurse cannot be effective in helping break through the denial. Although suggesting marriage counseling may be appropriate, it does not address the issue of denial. The comment "Why do you think that your wife is the cause of your problems?" enables the client to continue to avoid responsibility for his own behavior.

A client in the psychiatric hospital is attempting to communicate by stating, "Sky, flower, angry, green, opposite, blanket." The nurse recognizes what term as describing this type of communication? Multiple choice question Echolalia Word salad Confabulation Flight of ideas

Word salad is an incoherent mixture of words. Echolalia is a pathologic repetition of another's words or phrases. Confabulation is the unconscious filling in of memory gaps with imagined or untrue experiences. Flight of ideas is a speech pattern of rapid transition from topic to topic. The client's statement is too limited to be considered flight of ideas.

A 6-year-old child is in the acute phase of nephrotic syndrome. The mother asks the nurse about play activities for her child. What should the nurse suggest? Multiple selection question Hula hoop Video games Large puzzles Stuffed animal Children's books

video games and children books Age-appropriate video games do not require excessive energy to play and will help a 6-year-old child avoid boredom. Children's books are appropriate for 6-year-old children because at this age they are beginning to read. Also, the parents may read to the child. This activity does not require energy. Playing with a hula hoop requires energy that a child in the acute phase of nephrotic syndrome does not have. Large puzzles are more appropriate for toddlers, who are developing fine motor skills. A stuffed animal is more appropriate for an infant or toddler. It is a passive toy that will not be stimulating for a 6-year-old child.

A client is admitted to the acute psychiatric unit of the local community hospital. The client is guarded and suspicious. After a thorough evaluation, a diagnosis of schizophrenia, paranoid type, is made. What initial approach should be used by the nurse assigned to establish a therapeutic one-on-one relationship with this client? Multiple choice question Casual and honest Warm and friendly Permissive and distant Undemanding and watchful

Casual and honest Individuals with schizophrenia of the paranoid type are more apt to trust nurses who display matter-of-fact, predictable behaviors. The warm and friendly approach is too threatening to the individual with schizophrenia of the paranoid type, who does not trust others. The permissive and distant approach may be perceived as a lack of interest; these behaviors tend to reinforce a paranoid individual's social withdrawal. Watchful behavior on the part of the nurse reinforces a paranoid client's suspiciousness.

A client has undergone surgery with general anesthesia. Within how many hours after surgery should the nurse notify the primary healthcare provider if the client does not void? Multiple choice question 4 hours 8 hours 10 hours 12 hours

Clients should urinate 6 to 8 hours after catheter removal. Decreased bladder muscle tone results from the depressant effects of anesthesia and the handling of tissues and adjacent organs during surgery. Catheterization may be necessary to prevent overdistention of the bladder. Four hours may be too early to expect recovery from the depressant effects of anesthesia. Ten and 12 hours are too long to wait to call the primary healthcare provider. This length of time without voiding may result in overdistention of the bladder.

A nurse is interviewing a client newly admitted to an outpatient program after withdrawal from alcohol. What behavior best indicates that the client has accepted that drinking is a problem? Multiple choice question Participates in scheduled counseling sessions Attends Alcoholics Anonymous meetings daily Volunteers to be a sponsor for another alcoholic Apologizes to family members for causing distress

Daily attendance at AA meetings usually indicates an acceptance of the problem and a desire for help. Attendance at counseling sessions is helpful but is not specific to the problem of alcoholism. Clients with alcohol problems should not sponsor other clients until sobriety has been maintained for a long period. Clients with alcohol problems may say that they are sorry many times but still not take responsibility for their drinking problem

A nurse is caring for a client exhibiting compulsive behaviors. The nurse concludes that the compulsive behavior usually incorporates the use of which defense mechanism? Projection Regression Displacement Rationalization

Displacement Displacement is the unconscious redirection of an emotion from a threatening source to a nonthreatening source. Projection is the attribution of one's unacceptable feelings and thoughts to someone else. Regression is the return to an earlier, more comfortable level of behavior; it is a retreat from the present. Rationalization is the attempt to make unacceptable behavior or feelings acceptable by justifying the reasons for them.

What should the nurse should explain to the newly pregnant primigravida about how and when the fetal heartbeat will first be heard? Multiple choice question A fetoscope around 8 weeks A fetoscope at 12 to 14 weeks Electronic Doppler ultrasonography after 17 weeks Electronic Doppler ultrasonography at 10 to 12 weeks

Electronic Doppler ultrasonography at 10 to 12 weeks The fetal heartbeat can be heard on electronic Doppler ultrasound between 10 and 12 weeks' gestation. Around 8 weeks is too early for the heartbeat to be heard with a fetoscope; a fetoscope can pick up the fetal heartbeat accurately around the twentieth week. The fetal heartbeat can be heard at least 5 weeks earlier with the use of electronic Doppler ultrasound.

A nurse teaches a new mother about neonatal weight loss in the first 3 days of life. How does the nurse explain the cause of this weight loss? Multiple choice question An allergy to formula A hypoglycemic response Ineffective feeding techniques Excretion of accumulated excess fluids

Excretion of accumulated excess fluids Early weight loss occurs because excess fluid is lost, not body mass. Weight loss is expected; there are no data to support an allergic response. Weight loss is not related to hypoglycemia. Neither breast nor formula feeding will prevent the 10% weight loss that is expected in the first few days of life.

A nurse is assessing a client with major depression. Which clinical manifestation reflects a disturbance in affect related to depression? Multiple choice question Echolalia Delusions Confusion Hopelessness

Hopelessness Feelings of hopelessness are symptomatic of depression; the individual feels unable to find any solution to problems and therefore feels overwhelmed. Echolalia, the pathological meaningless repetition of another's words or phrases, is associated with schizophrenia, not with depression. Delusions are associated with psychotic disorders such as schizophrenia, not depression. Confusion is not common because these individuals are in contact with reality.

client with paralysis of the legs is found to have somatoform disorder, conversion type. What must the nurse consider when formulating a plan of care for this client? Multiple choice question The illness is very real to the client and requires appropriate nursing care. Although the client believes that there is an illness, there is no cause for concern. There is no physiological basis for the illness; therefore only emotional care is needed. Nursing intervention is needed even though the nurse understands that the client is not ill.

Individuals who have somatoform disorders are really ill; they need care in a nonthreatening environment. The client requires physiological and emotional care for treatment of motor or sensory functional deficits.

A woman who was discharged recently from the hospital after undergoing a hysterectomy calls the clinic and states that she has tenderness, redness, and swelling in her right calf. What priority action should the nurse instruct the client to take? Multiple choice question "Stay in bed for at least 3 days." "Keep the legs elevated while sitting." "Apply a warm compress to the affected calf twice a day." "Call an ambulance to go to the emergency department."

"Call an ambulance to go to the emergency department." The client's description of her problem is indicative of thrombophlebitis; this is a medical emergency because it may precipitate a pulmonary embolism. The client must be assessed by a primary healthcare provider. Intravenous anticoagulants will probably be necessary. Although bed rest may be prescribed eventually, a delay in pharmacologic treatment may jeopardize the client's status. Elevation of the legs may be prescribed eventually, after the thrombophlebitis is resolved. Although warm compresses are commonly prescribed, a delay in pharmacologic treatment may jeopardize the client's status.

While inspecting her newborn a mother asks the nurse whether her baby has flat feet. How should the nurse respond? Multiple choice question "Flat feet are more common in children than adults." "That's hard to assess because the feet are so small." "There may be a bone defect that needs further assessment." "Infants' feet appear flat because the arch is covered with a fat pad."

"Infants' feet appear flat because the arch is covered with a fat pad." A fat pad covers the arch in newborns and infants; the arch develops when the child begins to walk. Flat feet are no more common in children than in adults. The size of the feet is not relevant; arch development is related to walking. Flat feet are not associated with deformities of the bones.

The nurse is teaching an adolescent about the different methods of contraception. Which statement made by the adolescent indicates a need for further teaching? Multiple choice question A diaphragm is a soft rubber dome and requires nothing else. Condoms require consistent use and may cause decreased spontaneity. Lea's shield is less effective in women who have had previous deliveries. A cervical cap is contraindicated in women with a history of toxic shock syndrome.

A diaphragm is a soft rubber dome and requires nothing else. A diaphragm is a latex or rubber dome-shaped cup contraceptive device used along with spermicidal jelly to cover the cervical opening. A condom covers the penis and traps the sperm. Condoms need to be used consistently and may cause decreased spontaneity. Lea's shield is a reusable vaginal contraceptive that is elliptical in shape. However, it is not effective in women who have already delivered a baby, as the vagina does not remain elliptical. The cervical cap should not be used by women who have a history of toxic shock syndrome.

When working with a client who is in an alcohol detoxification program, what nursing action is most important? Multiple choice question Address the client's holistic needs. Support the client's need for nurture. Discuss with the client the negative effects of alcohol. Promote the client's compliance with the program through gentle prodding.

Address the client's holistic needs. Clients who abuse alcohol characteristically have multiple nursing care needs, among them physiological, psychological, social, and occupational. Although nurture is important, this client must learn self-reliance. Discussing with the client the negative effects of alcohol is probably an old story to this client and will have a minimal positive effect. Promoting the client's compliance with the program through gentle prodding will not provide an atmosphere that can help the client withstand the stress of the detoxification program.

A nurse, planning care for a client who is an alcoholic, knows that the most serious life-threatening effects of alcohol withdrawal usually begin after a specific time interval. How many hours after the last drink do they occur? Multiple choice question 8 to 12 12 to 24 24 to 72 72 to 96

Alcohol withdrawal delirium, a life-threatening central nervous system response to alcohol withdrawal, occurs in 1 to 3 days, when the blood alcohol level drops as alcohol is detoxified and excreted. Jitteriness, nervousness, and insomnia may occur 8 to 12 hours after withdrawal; these are not life-threatening issues. Nervousness, insomnia, nausea, vomiting, and increased blood pressure and pulse may occur after 12 to 24 hours; these are not life-threatening problems. Withdrawal symptoms will have begun to subside after 72 to 96 hours, and the risk for complications is diminished.

Which topic should the nurse include in the anticipatory guidance provided to the parents of a 7-year-old client during a scheduled health maintenance visit? Multiple choice question Erratic mood changes An increase in allergies Strong food preferences Changes that occur with puberty

An increase in allergies While few illnesses are expected for the 7-year-old client, an increase in allergies may occur; therefore, the nurse should include this topic in the anticipatory guidance provided to the client's parents. Erratic mood changes and strong food preferences occur more often for the 6-, not 7-year-old client. Changes that occur with puberty is more appropriate for the 11- to 12-year-old client, not the 7-year-old client.

What does the nurse identify as an early sign of chronic lead poisoning (plumbism) in school-aged children? Multiple choice question Anemia Seizures Cognitive impairment (CI) Renal calcium dysfunction

Anemia The bone marrow is most susceptible to lead toxicity. Interference with hemoglobin biosynthesis leads to early signs and symptoms of anemia. Seizure activity and CI are late responses indicating central nervous system involvement. Renal calcium dysfunction is a late response indicating kidney damage; loss of protein and other substances occurs first.

Two female clients on the psychiatric unit have become very much attached to each other and are found in bed together. They become angry and sarcastic when the nurse asks one of them to return to her own bed. How can the nurse best address this situation? Multiple choice question By asking the health care provider to transfer one of the clients to another unit By limiting their privileges for several days because their behavior is undesirable By adopting a matter-of-fact, nonjudgmental attitude and setting limits on the behavior By supervising them carefully and separating them when possible throughout the day and always at night

By adopting a matter-of-fact, nonjudgmental attitude and setting limits on the behavior Everyone has the right to his or her sexual orientation and preferences, but limits must be set on acting-out behavior on a psychiatric unit. Helping clients deal with their sexuality in a more appropriate manner is more therapeutic than continuous separation by the staff. Punishment is inappropriate.

An older adult who lives alone tells a nurse at the community health center, "I really don't need anyone to talk to. The TV is my best friend." What defense mechanism does the nurse identify? Multiple choice question Denial Projection Sublimation Displacement

Denial

One hour postpartum a nurse assesses the amount of vaginal bleeding and determines that a client's uterus has become relaxed and boggy. Which intervention is a priority for the nurse to take in this situation? Multiple choice question Massage the uterus until firm. Check the client's blood pressure. Obtain a prescription for oxytocin. Notify the primary healthcare provider immediately.

Immediate action to prevent excessive bleeding involves massaging the fundus until it is firm, which stimulates uterine muscle contraction. Obtaining the blood pressure is indicated if the bleeding persists. Obtaining a prescription for oxytocin may not be necessary if fundal massage is effective. Notifying the primary healthcare provider immediately is not necessary unless bleeding persists after uterine massage.

A client who was admitted to the psychiatric unit because of a major depressive disorder is exhibiting increasingly withdrawn behavior. The nurse understands that eventually the client will experience what feelings? Multiple choice question Hedonia Isolation Paranoia Ambivalence

In an attempt to control anxiety, the client continues to retreat from people and the activities within the environment; this will eventually precipitate feelings of loneliness and isolation. Depressed clients exhibit a decreased interest in pleasurable activities (anhedonia) rather than an excessive interest in pleasurable activities (hedonia). Paranoia may be a cause, not a result, of withdrawal. Ambivalence, or experiencing conflicting emotions at the same time, is not precipitated by depression.

A client who is to begin a physical therapy regimen after orthopedic surgery expresses anxiety about starting this new therapy. Why does the nurse respond that some of this apprehension can be an asset? Multiple choice question Slow physiological function Increase alertness to the environment Mobilize automatic behavioral responses Promote the use of ego defense mechanisms .

Mild and moderate levels of anxiety can be beneficial because they focus attention on the environment by attempting to ward off additional anxiety. Initially anxiety amplifies physiological function; function decreases after prolonged anxiety because of exhaustion. Automatic behavioral responses and ego defense mechanisms may hinder, rather than increase, an individual's awareness

During an assessment interview the client reports overwhelming, irresistible attacks of sleep. Which sleep disorder does the nurse conclude that the client is experiencing? Multiple choice question Insomnia Narcolepsy Sleep terror Sleep apnea

Narcolepsy is overwhelming sleepiness that results in irresistible attacks of sleep, loss of muscle tone (cataplexy), and hallucinations or sleep paralysis at the beginning or end of sleep episodes; the person usually awakens from the sleep feeling refreshed. Insomnia is difficulty initiating or maintaining sleep. Sleep terrors are recurrent episodes of abrupt awakening from sleep accompanied by intense fear, screaming, tachycardia, tachypnea, and diaphoresis with no detailed dream recall. Sleep apnea is a breathing-related sleep disorder caused by disrupted respirations or airway obstruction; sleep is disrupted numerous times throughout the night.

A nurse enters a depressed client's room on the evening of admission and observes the client sitting in a chair crying. What is the most therapeutic response by the nurse? Multiple choice question "You're crying. Let's talk about it." "Let me get a cup of coffee; then we can talk." "Visitors will be here soon; you'd better get ready." "You'll feel better soon. Come to the sitting room with me."

Noting that the client is crying and suggesting that the nurse and client talk about it addresses the behavior observed, and the offer by the nurse to spend time to help the client implies that the client is worthy. With "Let me get a cup of coffee; then we can talk" the nurse offers to help but places the client second by stating the desire to get coffee first. The nurse denies the client's feelings by focusing on getting ready for visitors. Assuring the client that the client will feel better soon and asking the client to come to the sitting room constitutes false reassurance. The nurse first recognizes the client's feelings and then moves away from discussing them.

A pregnant client with severe abdominal pain and heavy bleeding is being prepared for a cesarean birth. What is the priority medical intervention? Multiple choice question Teaching coughing and deep-breathing techniques Sterilizing the surgical site and administering an enema Providing a sterile gown and inserting an indwelling catheter Obtaining informed consent and assessing the client for drug allergies

Obtaining informed consent and assessing the client for drug allergies In an emergency surgical situation when invasive techniques are necessary, it is important to have a signed consent on file as well as a history of the client's known allergies. Teaching coughing and deep-breathing techniques is not a priority in an emergency such as this. In an emergency, sterilization of the surgical site is performed in the operating room; an enema usually is not given before a cesarean, especially to a client who is bleeding, because it may stimulate contractions and worsen the hemorrhage.

A newborn is admitted to the nursery. The newborn weighs 10 lb, 2 oz (4592 g), which is 2 lb (907 g) more than the birthweight of any of the neonate's siblings. Which intervention should the nurse implement in relation to this baby's birth weight? Multiple choice question Document the findings Delay starting oral feedings Perform serial glucose readings Place the newborn in a heated crib

Perform serial glucose readings A large newborn may be the result of gestational diabetes; it is necessary to check the neonate for hypoglycemia, because maternal glucose is no longer available. The nurse should do more than document the findings; the primary healthcare provider should be notified after the serial glucose readings are taken. Placing the infant in a heated crib is indicated if the temperature is low and the newborn needs additional warmth. The infant may be hypoglycemic and require the glucose in an oral feeding immediately.

The nurse understands that paranoid delusions may be related to which defense mechanism? Multiple choice question Projection Regression Repression Identification

Projection Projection is a mechanism in which inner thoughts and feelings are projected onto the environment, seeming to come from outside the self rather than from within. Regression is the use of a behavioral characteristic appropriate to an earlier level of development. Repression is the involuntary exclusion of painful or conflicting thoughts from awareness. Identification is taking on the thoughts and mannerisms of an individual who is admired or idealized.

A child is frightened and refuses to use the toilet after watching a television commercial that shows the toilet bowl turning into a monster and swallowing the person using it. Which behavior does the child exhibit? Multiple choice question Animism Modeling Aggression Desensitization

Relating lifelike qualities to nonliving things is called animism. Therefore, when a child believes that the toilet bowl turns into a monster and swallows a person, he or she exhibits animism. Aggression is characterized by an attempt to hurt a person or damage property. Modeling is associated with imitation of others' behavior. Desensitization is a therapeutic technique that involves exposing the child to the feared object in a safe situation.

After a large-scale disaster event, a client reports a feeling of numbness for 1 month. On assessment, the nurse notes a high score of all subscales of the impact of event scale—revised (IES-R) tool. Which healthcare professional should the nurse refer the client to for further evaluation? Multiple choice question Psychiatrist Triage officer Social worker Mental health counselor

Reporting a feeling of numbness more than 2 weeks after a disaster event indicates risk of post-traumatic stress disorder (PTSD). The client should be assessed using the impact of event scale—revised (IES-R) tool and should be referred to a psychiatrist if the scores on all subscales are high. The triage officer evaluates each client to determine priorities for treatment. The client is referred to a social worker or mental health counselor for counseling if any one subscale score of IES-R is high.

A 20-year-old homeless client at 38 weeks' gestation visits the prenatal clinic for the first time. She is accompanied by her 21-year-old boyfriend, who is the father of the baby. The nurse becomes concerned because as they sit in the waiting room, they are sneezing and yawning and have teary eyes. With what substance are these withdrawal signs associated? Multiple choice question Heroin Cocaine Morphine Phenobarbital

Research indicates that sneezing, yawning, and teary eyes are the first physical signs of withdrawal from heroin. Depression and irritability accompany withdrawal from cocaine. Restlessness, shakiness, hallucinations, and sometimes coma accompany withdrawal from morphine. Insomnia, seizures, weakness, sweating, and anxiety accompany withdrawal from phenobarbital.

What is the most appropriate long-term goal for a client experiencing dysfunctional grieving after the death of a spouse? Multiple choice question Resuming previously enjoyed activities Eating at least two meals a day with another person Decreasing negativistic thinking about other people Relocating to a state in which other family members reside

Resuming previously enjoyed activities Resuming previously enjoyed activities is realistic, specific, and measurable; it relates to the client's acceptance of a new reason for being. Eating at least two meals a day with another person may be an unrealistic goal. There are no data to indicate that the client is thinking negatively about others. Relocating to a state in which other family members reside may be an unrealistic goal, or the client may not want to do this.

What should the nurse suggest when parents ask what to do about their preschooler's stuttering? Multiple choice question Speak clearly and do not complete the child's sentences. Avoid looking at the child when he experiences difficulty forming words. Help the child by supplying the correct word when he experiences a block. Stop the conversation and tell the child to speak slowly when starting again.

Speak clearly and do not complete the child's sentences. During the preschool years speech dysfluency is a typical characteristic of language development; it will resolve if the child is spoken to clearly and is not corrected. Avoiding eye contact, supplying the correct word, or drawing attention to the stuttering is demeaning ; it may decrease the child's self-esteem and worsen the stuttering.

A nursing instructor asks a nursing student about the sleep pattern of teenagers. Which statements made by the student indicate adequate learning? Multiple selection question Teenagers often have reduced hours of sleep. Teenagers often suffer from restless leg syndrome. Teenagers get roughly 7.5 hours of sleep each night. Twenty percent of a teen's sleep cycle is rapid eye movement (REM) sleep. Teenagers resist sleeping because they are unaware of fatigue.

Teenagers often have reduced hours of sleep. Teenagers get roughly 7.5 hours of sleep each night. The typical adolescent is subject to a number of changes, such as school demands, after-school social activities, and part-time jobs, that reduce the time spent sleeping. On average, teenagers get about 7.5 hours of sleep per night; preschoolers sleep an average 12 hours a night. Restless leg syndrome is common in young adults, not teenagers. Young adults, not adolescents, have 20% of their sleep time in REM sleep. Preschoolers often resist sleeping because they are unaware of fatigue or have a need to be independenT

what ratio would a mother get a Maternal rubella vaccination?

anything less than 1:8 rubella titer

According to Kohlberg's theory of moral development, which statement about conventional reasoning is true? Multiple choice question A person wants to fulfill family expectations. A child's thinking is mostly based on likes and pleasures. A child recognizes that there is more than one correct view. An individual moves away from moral decisions based on authority.

A person wants to fulfill family expectations.

Which intervention will the nurse implement when assisting a child with a history of aggressive behavior to regain control in the triggering phase of an assault cycle? Multiple choice question Discuss alternative behaviors to substitute for aggression. Provide the child with a quiet, low-stimulus environment. Speak to the child in a calm but firm manner. Administer medication as needed (PRN) to facilitate de-escalation.

In the triggering phase, the client's behavior is nonthreatening and poses no danger to others. Minimizing environmental stimuli and providing a calm, nonthreatening environment likely will serve to help the client de-escalate and regain control. Discussion of substitute behaviors is effective only once the crisis is over (postcrisis phase). As the client escalates, the nurse needs to begin to assume control by presenting a calm but firm tone of voice and demeanor. It is at this time that appropriate oral PRN medications may be helpful.

A client at 10 weeks' gestation phones the prenatal clinic to report that she is experiencing some vaginal bleeding and abdominal cramping. The nurse arranges for her to go to the local hospital. The vaginal examination reveals that her cervix is dilated 2 cm. What diagnosis should the nurse expect? Multiple choice question Septic abortion Inevitable abortion Threatened abortion Incomplete abortion

Inevitable abortion Once cervical dilation has begun, the abortion is classified as inevitable. In septic abortion the cervix is dilated and there is bleeding, but malodorous discharge is also present. Bleeding and cramping may be present in a threatened abortion; however, the cervix is still closed. The products of conception have been partially expelled in an incomplete abortion.

An adolescent client with antisocial personality disorder plans to live with the parents after discharge. The parents request advice on how to respond to their child's unruly behavior. What is the most therapeutic response by the nurse? Multiple choice question "Discuss the behavior with your child and encourage the development of self-control." "Avoid setting expectations for behavior and react to each situation as it arises." "Help your child find new friends and encourage finding a job and assuming personal responsibility." "Set clear limits, explain the consequences if your child disregards them, and firmly and consistently apply them."

Setting clear limits, explaining the consequences of disregarding them, and firmly and consistently applying them are the most therapeutic parental actions; the client must be made accountable for behavior and must know that manipulation and acting-out will not be tolerated. The response "Discuss the behavior with your child and encourage the development of self-control" is probably a continuation of the parents' previous response to the client and is of little value. The response "Avoid setting expectations for behavior and react to each situation as it arises" may cause the client to continue to act out to test the limit of the parents' endurance. The activities in the response "Help your child find new friends and encourage finding a job and assuming personal responsibility" are outside the parents' control.

While assessing a neonate, the nurse observes ecchymotic-appearing areas on the buttocks and sacrum. What does the nurse conclude that this discoloration is probably related to? Multiple choice question Skin color Gestational age Tendency to bleed Vaginal breech birth

Skin color These bluish discolorations are known as Mongolian spots, which are commonly found on the back and buttocks of dark-skinned newborns. These spots are unrelated to gestational age. Bluish spots on the buttocks are not areas of ecchymosis caused by bleeding. The buttocks and genitals of infants who are born vaginally in the breech presentation are usually edematous.

What is a major recognizable difference between anorexia nervosa clients and bulimia nervosa clients? Multiple choice question Anorexia nervosa clients tend to be more extroverted than clients with bulimia. Anorexia nervosa clients seek intimate relationships, whereas clients with bulimia avoid them. Anorexia nervosa clients are at greater risk for fluid and electrolyte imbalances than are clients with bulimia. Anorexia nervosa clients deny the problem, whereas clients with bulimia generally recognize that their eating pattern is abnormal.

The client with anorexia nervosa denies the illness; the client with bulimia nervosa hides the behavior because she recognizes that the behavior is a problem. Clients with anorexia nervosa are more introverted and tend to avoid relationships. Clients with bulimia are at a greater risk for fluid and electrolyte problems because of the purging; clients with anorexia nervosa are at greater risk for severe nutritional deficiencies.

A male client in a mental health facility turns his head to the side during a unit meeting as if he hears something. When the nurse comments about it, the client replies, "You know, it's that microcomputer those foreign agents implanted in my ear." In light of this statement, what does the nurse determine that the client is experiencing? Multiple choice question Illusions Delusional thoughts Neologistic thinking Disorganized cognition

The client's statement reveals the cognitive disturbance called a delusion, which is a fixed set of false beliefs that cannot be corrected by reason. An illusion is a misperception of an actual environmental stimulus. Disorganized thought would include the inability to organize thought process. Neologisms are made-up words understood only by the speaker.

A child has been found to have acute myelogenous leukemia. The practitioner has discussed the diagnosis and prognosis with the parents. Later, after visiting their child, the parents have a bitter argument. The nurse identifies what defense mechanism? Multiple choice question Denial Projection Displacement Compensation

The parents are focusing their feelings about their child's prognosis on someone or something else—in this case, each other. Denial is ignoring, avoiding, or refusing to recognize painful realities. Projection is the attribution of one's own feelings to another person. Compensation is making up for a perceived deficiency by emphasizing another feature perceived as an asset.

A nurse is caring for a client with the diagnosis of schizophrenia. What is a common problem for clients with this diagnosis? Multiple choice question Chronic confusion Disordered thinking Defined personal boundaries Violence directed toward others

The schizophrenic individual has neurobiological changes that cause disorders in thought process and perceiving reality. Chronic confusion and disorientation are not usually associated with this disorder. Illogical thinking and impaired judgment are associated with schizophrenia. Individuals with the diagnosis of schizophrenia often have personal boundary difficulties. They lack a sense of where their bodies end in relation to where others begin. Loss of ego boundaries can result in depersonalization and derealization. Most clients with schizophrenic disorders are not violent.

Which growth and developmental assessments should the nurse include when conducting a health maintenance visit for a 15-month-old toddler? Multiple selection question Weight Heart rate Blood pressure head circumference Developmental screening

Weight n head circumerence Growth and developmental assessments that the nurse should perform for a 15-month-old toddler include weight and head circumference. Data related to heart rate and blood pressure are vital signs and not growth and developmental assessments. A developmental surveillance, not screening, is appropriate for a 15-month-old toddler.

At times a client's anxiety level is so high that it blocks attempts at communication and the nurse is unsure of what is being said. To clarify understanding, the nurse says, "Let's see whether we mean the same thing." What communication technique is being used by the nurse? Multiple choice question Reflecting feelings Making observations Seeking consensual validation Attempting to place events in sequence

Seeking consensual validation is a technique that prevents misunderstanding so that both the client and the nurse can work toward a common goal in the therapeutic relationship. Reflection of feelings is used to increase client awareness but should not be used when the nurse is unsure of what the client is saying. Making observations refers more to nonverbal than to verbal communication. Placing events in a sequence helps organize content, but ideas should be clarified first by means of validation if the nurse is unsure of the meaning of what is being said.

Which statement by the nurse is true for collecting a urine sample in toddlers? Multiple choice question A hat is placed under the toilet seat. Urine can be squeezed from the diaper. Force the child to void in the unfamiliar receptacle. Single-use bags can be placed over the child's urethral meatus.

Single-use bags are placed over the child's urethral meatus for collecting urine in toddlers. A potty chair or specimen hat placed under the toilet seat is usually effective in cases of young children. Specimens obtained by squeezing urine from the diaper are not used because the results will be inaccurate. A young child is often reluctant to void in unfamiliar receptacles. They should not be forced to void.

A deeply depressed, withdrawn client remains curled up in bed and refuses to talk to the nurse. What should the nurse do initially to break through the client's withdrawal? Multiple choice question Sit with the client for set periods each hour. Touch the client gently on the arm when the opportunity arises. Urge the client to participate in simple games with other clients. Inform the client that going to the lounge is required in the daytime.

Sit with the client for set periods each hour. Sitting quietly with a severely withdrawn client can provide an opportunity for nonthreatening interaction. Entering a withdrawn client's body space is intrusive and stressful; it often precipitates a need for further withdrawal. The client is unable to socialize with others at this time. Placing demands on the withdrawn client causes a sense of threat, increased anxiety, and a need for additional withdrawal.

A nurse is assessing a client and attempting to distinguish between dementia and delirium. Which factors are unique to delirium? Multiple selection question Slurred speech Lability of mood Long-term memory loss Visual or tactile hallucinations Insidious deterioration of cognition A fluctuating level of consciousness

Slurred speech, visual/tactile hallucinations, n changing level of consciousness Delirium, a transient cognitive disorder caused by global dysfunction in cerebral metabolism, results in sparse or rapid speech that may be slurred and incoherent. Visual or tactile hallucinations and illusions may occur with delirium because of altered cerebral function; hallucinations are not prominent with dementia. Clients with delirium fluctuate from hyperalert to difficult to arouse; they may lose orientation to time and place. Clients with dementia do not have a fluctuating level of consciousness, but they may be confused and disoriented. Clients with delirium are consistently irritable, anxious, and fearful; lability of mood is associated with dementia. Short-term memory loss is associated with both delirium and dementia; eventually long-term memory loss is associated with dementia. The onset of delirium is abrupt (hours to days) and has an organic basis; it is often precipitated by drugs such as anesthesia, analgesics, and antibiotics or by conditions such as infections, end-stage kidney disease, and substance abuse or withdrawal. The onset of dementia is slow and insidious (year

A mother and her 5-year-old daughter have been referred to a child advocacy center for a forensic pediatric sexual examination. Before the child is examined or interviewed, the mother gives a detailed history, relaying her suspicion that the child's maternal grandfather sexually assaulted her. As the interview progresses, the mother suddenly says, "My father sexually molested me when I was a child, but I try not to think about it." What defense mechanism does the nurse recognize that the mother's statement demonstrates? Multiple choice question Introjection Suppression Reaction formation Passive aggression

Suppression is voluntary refusal to admit an unacceptable idea or behavior. Introjection is the unconscious incorporation of wishes, values, and attitudes of others as if they were one's own. Passive-aggressive behavior is the expression of anger and hostility toward others in an indirect and nonassertive way. Reaction formation is the exact opposite of an unconscious feeling.

A 14-year-old emancipated minor at 22 weeks' gestation comes in for her second prenatal examination. As she enters the examination room with her mother, she tells the nurse that she does not want her mother present for the examination. How should the nurse respond? Multiple choice question "Your mother needs to be present for the examination." "What's the problem with your mother being present?" "I'm sure that your mother wants to be with you for support." Tell the mother, "I'm sorry, but I need to ask you to stay in the waiting area."

Tell the mother, "I'm sorry, but I need to ask you to stay in the waiting area." In many jurisdictions a minor who is self-supporting and living away from home, providing military service, married, pregnant, or a parent is considered an emancipated minor. The emancipated minor assumes most responsibilities before the age of 18 years. An emancipated minor is entitled to confidentiality in dealings with healthcare providers. Therefore it is appropriate to ask the client's mother to step out of the room.

The nurse is teaching a client about self-management to prevent dry skin. Which statement made by the client indicates the need for further teaching? Multiple choice question "I should use nonalkaline soap for a bath." "I should apply rubbing alcohol to the skin." "I should avoid clothing that continuously rubs the skin." "I should use a room humidifier during the winter months."

To prevent dry skin, rubbing alcohol is contraindicated because alcohol increases skin dryness. Use of nonalkaline soap for bathing prevents dry skin. Avoid clothing that continuously rubs the skin such as tight belts and nylon stockings. Use room humidifiers during winter months because skin is drier in winter.

The nurse is assessing her assignment of four postpartum clients. Which conditions increase the risk for postpartum hemorrhage? Multiple selection question Twin birth Overdistended bladder Hypertonic uterine dystocia Retained placental fragments Mild gestational hypertension

Twin birth, over distended bladder, n retained placental fragments Overdistention of the uterus may lead to delayed or inadequate uterine contractions. An overdistended bladder may inhibit uterine contractions. Retained placental fragments inhibit uterine contractions. Clients with ineffective uterine contractions are treated with rest and sedatives; although labor is prolonged, postpartum hemorrhage is not expected. Mild gestational hypertension does not interfere with uterine involution.

A primary healthcare provider prescribes a diagnostic workup for a client who may have myasthenia gravis. What is the initial nursing goal for the client during the diagnostic phase? Multiple choice question Adhere to a teaching plan. Achieve psychologic adjustment. Maintain present muscle strength. Prepare for the development of myasthenic crisis.

Until the diagnosis is confirmed, the primary goal should be to maintain adequate activity and prevent muscle atrophy. It is too early to develop a teaching plan; the diagnosis is not yet established. It is too early to achieve psychologic adjustment; the client cannot adjust if a diagnosis is not yet confirmed. Preparing for the development of myasthenic crisis is not a goal.

A school nurse is teaching a high school health class about inhalant abuse. What serious effect of using inhalants should the nurse discuss? Multiple choice question Esophageal varices Acute electrolyte imbalances Extrapyramidal tract symptoms Death in one third of first-time users

Use of inhalants, called "huffing," is most often seen in preadolescent males in rural areas, and it can be lethal in overdose. Esophageal varices are associated with alcoholic cirrhosis. Acute electrolyte imbalances are associated with alcoholic cirrhosis and are related to malnutrition, dehydration, and ascites. Extrapyramidal tract symptoms are associated with typical antipsychotic medications.

Elbow restraints are prescribed for an 18-month-old toddler who just had surgery for a cleft palate. The nurse explains to the parents that the restraints are used to keep the child from doing what? Multiple choice question Playing with unsterile toys Rolling to a supine position Putting fingers into the mouth Removing the nasogastric tube

putting fingers into mouth The suture lines in the mouth must be protected. Because the toddler uses the mouth to explore the environment, elbow restraints are needed to keep the child from placing fingers or objects in the mouth. The child should have time to play with toys, but with supervision to prevent mouthing activities that could disrupt the suture line. The supine position is acceptable; the toddler should be able to move freely when asleep. A nasogastric tube is not used.

While a nurse is assisting with morning care for a client with the diagnosis of schizophrenia, the client suddenly throws off the covers and starts shouting, "My body is disintegrating! I'm being pinched." What term best describes the client's behavior? Multiple choice question Somatic delusion Paranoid ideation Loose association Ideas of reference

A somatic delusion is a false feeling about the physical self that is caused by a loss of reality testing. Paranoid ideations are beliefs that the individual is being singled out for unfair treatment. Loose associations are verbalizations that are difficult to understand because the links between thoughts are not apparent. Ideas of reference are false beliefs that the words and actions of others are concerned with or are directed toward the individual.

A preschool child is found to have atopic dermatitis. The nurse emphasizes that the child should be discouraged from scratching. The child's mother asks why scratching should be prevented. What is the nurse's response? Multiple choice question "Scratching causes lesions to become more contagious." "Scratching spreads dermatitis to other areas of the body." "Scratching results in skin breaks that can lead to infection." "Scratching produces changes that are precursors to skin cancer."

"Scratching results in skin breaks that can lead to infection." Scratching can break the integrity of the skin, leaving it vulnerable to infection. Dermatitis is a response to an allergen; it is not contagious. Scratching will not cause the dermatitis to spread. There are no data to indicate that scratching or dermatitis is a precursor to skin cancer.

An 8-year-old child with cerebral palsy is admitted to the hospital for a tendon-lengthening procedure. After the surgery the parents ask a nurse why their child must wear braces and shoes for at least 12 hours a day, even while in bed. What is the best response by the nurse? Multiple choice question "Ambulation should be encouraged as soon as possible." "They maintain body alignment and help prevent foot drop." "They stretch your child's ligaments and strengthen muscle tone." "It helps your child accept the physical constraints of the condition."

"They maintain body alignment and help prevent foot drop." Braces are worn to enable the spastic child to control movement. They also prevent deformities that can occur as a result of misalignment. Early ambulation is promoted by maintaining muscle strength and tone, but it is not the reason for applying braces. Exercises, not braces, are used to stretch ligaments and improve muscle strength and tone. Promoting acceptance is not the purpose of braces and shoes. The child is in Erikson's stage of industry versus inferiority, and the braces and shoes will promote independence.

A 16-year-old adolescent at 24 weeks' gestation visits the prenatal clinic for the first time. After the physical examination she tells the nurse, "I can't believe how big I am. Will I get much bigger?" What information about adolescent growth and development does the nurse need to understand before responding? Multiple choice question Adolescents generally regain their figures 2 weeks after the birth, so size is of moderate concern. Adolescents are in a high-risk category, so weight gain should be limited to prevent complications. Body image is very important to adolescents; therefore pregnant teenagers are overly concerned about body size. Physiological growth in adolescents is more rapid than in adults, so the gravid size is larger than that of an adult woman.

Body image is very important to adolescents; therefore pregnant teenagers are overly concerned about body size. Because of the changes in body size, the pregnant adolescent may feel insecure as she struggles to establish her identity. There are no data to support the statement that adolescents generally regain their figures 2 weeks after the birth. The optimal weight gain for an adolescent is at the upper range for her body mass index; this will help prevent complications. Although physiological growth is rapid, the adolescent's gravid size falls within the expected parameters for pregnant women.

A 3-month-old infant with severe developmental dysplasia of the hip has a hip spica cast applied. Which instruction should the nurse give the parents to help prevent a serious complication? Multiple choice question Change diapers frequently. Decrease the number of feedings per day. Call the primary healthcare provider if a foul smell is detected. Avoid turning the child from the prone to the supine position.

Call the primary healthcare provider if a foul smell is detected. A foul smell emanating from the cast may indicate the development of an infection and necessitates immediate intervention to prevent sepsis, which is a serious complication involving tissue infection that has worsened to become a bloodstream infection. Soiling of the cast with excreta, although problematic, is not a serious complication. Decreasing the number of feedings per day is not necessary or desirable. The infant's position should be changed frequently.

A client experiencing nonspecific, excessive, unpleasant feelings of being worried concerning one's safety likely is experiencing which mental health disorder? Multiple choice question Phobia Panic disorder Generalized anxiety disorder (GAD) Posttraumatic stress disorder (PTSD)

Generalized anxiety disorder is the manifestation of both physical and cognitive symptoms of chronic or excessive anxiety/worry. A phobia is a fear of a specific type of stimuli. Panic is an extreme stage of anxiety. Posttraumatic stress disorder is a state of anxiety resulting from a traumatic experience.

A young adult client is admitted to the hospital with a diagnosis of schizophrenia, paranoid type. The client has been saying, "The voices in heaven are telling me to come home to God." What should initial nursing care be focused on? Multiple choice question Disturbed self-esteem Potential for self-harm Dysfunctional verbal communication Impaired perception of environmental stimuli

Potential for self-harm Client safety always is the priority over any other client need, and command hallucinations increase the risk of injury. Although promoting self-esteem is important, this is not a priority at this time. There are no data to support the need to focus on the client's ability to verbally communicate. Verbal hallucinations occur within the individual; they are not precipitated by an environmental stimulus.

What nursing intervention should be implemented routinely after a client has a vacuum aspiration abortion? Multiple choice question Giving the client the prescribed oxytocic medication Preparing the client for discharge within 30 minutes Teaching the client about the various methods of birth control Encouraging the client to take the prescribed antibiotic medication

Prophylactic antibiotics after a vacuum extraction abortion decrease the incidence of infection. Oxytocics are not used routinely after an abortion unless there is excessive vaginal bleeding. The client is usually observed for 1 to 3 hours before being discharged. Birth control instructions should be given before the abortion; a client is not receptive to teaching immediately after the procedure.

If the nurse is considering whether the right equipment and resources are available to complete a task, which delegation right is considered? Multiple choice question Task Supervision Circumstance Communication

If the right equipment and resources are available to complete a task, it is considered the right circumstance. Task is the delegation right that involves asking if the task is appropriate to delegate based on institutional policies and procedures. Supervision is the delegation right involving the provision of clear feedback related to completion of an assigned task. Communication is the right that involves asking the delegator and delegate to understand a common work-related language.

A nurse is planning to teach a 9-year-old child with type 1 diabetes to perform blood glucose monitoring. What information does the nurse use as a basis for the teaching? Multiple choice question A child this age is too young to learn this skill. Learning progresses on a continual line upward. There must be adequate teaching for the child to learn. Life experiences are limited because of the child's age.

Life experiences are limited because of the child's age. Many behaviors require a background of knowledge, skills, and attitudes (experiential readiness); if this background is limited, more learning must take place before the child can learn a new skill. A 9-year-old child has the cognitive ability to learn how to self-monitor the blood glucose level. Learning progress varies according to the individual; there are progressive and regressive periods. Although adequate teaching may facilitate learning, other factors such as readiness for learning and a willingness to learn affect whether learning occurs.

The nurse is providing education to the parents of a preschool-age client who is experiencing a severe fear of the dark. Which treatment option should the nurse share with the parents during the teaching session? Multiple choice question Prescription medication Electroconvulsive therapy Intensive therapy sessions Repetition of brave statements

Repetition of brave statements Repetition of brave statements is a treatment option that the nurse should share with the parents of a preschool-age client who experiences severe fear of the dark. Prescription medication, electroconvulsive therapy, and intensive therapy sessions are not appropriate treatment options for the nurse to share with this client's parents.

A 23-year-old primigravida is at her first prenatal appointment today. Ultrasound indicates that she is at 9 weeks' gestation. She asks when she can first expect to feel her baby move. What is the best response by the nurse? Multiple choice question "You should be able to feel the baby move any day now." "You should feel your first light movement of the baby around 24 weeks." "Most women can first detect movement of their babies by 12 to 14 weeks." "Many women are able to first feel light movement between 18 and 20 weeks."

"Many women are able to first feel light movement between 18 and 20 weeks." Fetal movement can be felt after 18 weeks and usually by 20 weeks in a primigravida. Fetal movement is normally not felt before 18 weeks' gestation, when the uterus has risen into the abdomen. Fetal movement should continue to be felt at 24 weeks' gestation, but normally is felt 4 to 6 weeks before this time.

A nurse is working with a married woman who has come to the emergency department several times with injuries that appear to be related to domestic violence. While talking with the nurse manager, the nurse expresses disgust that the woman keeps returning to the situation. What is the best response by the nurse manager? Multiple choice question "She must not have the financial resources to leave her husband." "Most women try to leave about six times before they are successful." "There's nothing the staff can do; people are free to choose their own lives." "These women should be told how stupid they are to stay in that kind of situation."

"Most women try to leave about six times before they are successful." Nurses who work with victims of partner abuse need to be supportive and patient. It takes time and several attempts for most victims to leave abusive relationships. It may or may not be true that the client does not have the financial resources to leave her husband; there is not enough information to support this conclusion. The staff can encourage the woman to make plans for addressing various potential events and provide information about social services and telephone help lines. Shaming women in this position will simply make them less likely to seek help.

A nurse finds that a mother severely punishes her two-year-old child. What should the nurse advise the mother, according to the Erikson's theory? Multiple choice question "Your behavior may induce a feeling of isolation in your child." "You need to establish a sense of trust or the child may lose trust in you." "Your child needs support and love or else he or she may develop feelings of shame and doubt." "You should control your child's impulses or the child may suffer guilt and frustration."

"Your child needs support and love or else he or she may develop feelings of shame and doubt." According to Erikson, a two-year-old child is in the stage of "autonomy versus sense of shame and doubt." The nurse should counsel the mother that severely punishing her child may lead to a feeling of shame and doubt in the child. This may hamper the child's healthy growth. Intimacy versus isolation stage leads to feelings of isolation in young adults. A feeling of mistrust is observed in infants less than one year of age. Guilt and frustration is observed in preschoolers in the stage of initiative versus guilt.

What is the nurse's most important concern when caring for a client with a ruptured tubal pregnancy? Multiple choice question Infection Hypervolemia Protein deficiency Diminished cardiac output

Diminished cardiac output Uncontrolled bleeding causes decreased circulating blood volume, and therefore there is a decreased cardiac output. Infection may occur later but is not a problem at this time. There will be hypovolemia, not hypervolemia, because of a decrease in circulating blood volume resulting from hemorrhage. There are no data to justify the conclusion that the client has a protein deficiency.

A nurse is caring for four mother-baby couplets on the postpartum unit. Which new mother is at the greatest risk for postpartum hemorrhage? Multiple choice question A primipara who has given birth to an 8-lb baby A grand multipara who experienced a labor that lasted 1 hour A multipara whose placental separation occurred 10 minutes after she gave birth A primipara who received epidural anesthesia throughout the birthing experience

A grand multipara who experienced a labor that lasted 1 hour Increased parity contributes to an increased incidence of uterine atony because the uterine muscle may not contract effectively, leading to postpartum hemorrhage; it is not uncommon for a grand multipara to have a labor that lasts 1 hour. A primipara should maintain a well-contracted uterus; with only one pregnancy, the uterus usually maintains its tone. Expulsion of the placenta 10 minutes after the birth of the fetus is expected and will not affect the tone of the uterus. Uterine atony is not a major problem associated with epidural anesthesia.

A client on the psychiatric unit is noisy, loud, and disruptive. The nurse informs the client, "Unless you're quiet, you'll be isolated and put in restraints, if necessary." How can this interaction be described in relation to the law? Multiple choice question The information given to the client is actually assault. The client's behavior is to be expected and should be ignored. Clients who are hyperactive need to be restrained for their own protection. Clients who are disruptive and hyperactive cannot be expected to understand instructions.

A threat is considered a type of assault (legally, an intentional tort). The client's behavior may be expected, but it should be dealt with directly; behavior should never be ignored. Restraints are unnecessary for this client; they are used only when the client's behavior escalates to the point that it becomes a threat to the client or others. Disruptive, hyperactive clients may respond to calm limit-setting.

The nurse is reviewing the data of female clients with amenorrhea. Which client may be diagnosed with primary amenorrhea based on the given data? Multiple choice question A 13-year-old with undeveloped secondary sexual characteristics who has not yet reached menarche An 18-year-old with normally developed secondary sexual characteristics who has not yet reached menarche A 17-year-old who is in her first trimester of pregnancy A 20-year-old triathlete whose menstruation ceased three months ago

An 18-year-old with normally developed secondary sexual characteristics who has not yet reached menarche Absence of menstrual flow is termed amenorrhea. This is a clinical sign for a variety of disorders. The absence of menses by age 16.5 years, regardless of normal growth and development, is known as primary amenorrhea, so the 18-year-old with normal development of secondary sexual characteristics who has not yet started menstruating is experiencing primary amenorrhea. The 13-year-old has not yet started menstruating, but her secondary sexual characteristics have not yet begun developing, and so she is not yet said to be experiencing primary amenorrhea. The 17-year-old is experiencing secondary amenorrhea since her lack of menstruation is due to her pregnancy. The 20-year-old triathlete is also said to be experiencing amenorrhea that is secondary because it is likely due to her rigorous physical activity.

A nursing instructor asks a nursing student about the physical changes observed in toddlers. Which statement made by the student indicates the need for further teaching? Multiple choice question "The average toddler grows 4.2 cm each year." "There is a rapid development of motor skills." "Locomotion skills include running, jumping, and kicking." "Average toddlers gain 5 to 7 pounds (2.3 to 3.2 kg) each year."

An average toddler grows 6.2 cm each year. Hence, when the student says that the average growth is 4.2 cm, it indicates the need for further teaching. There is a rapid development of motor skills in children, which enables them to perform self-care activities like feeding, dressing, and toileting. Locomotion skills that develop at this age include running, jumping, standing on one foot for several seconds, and kicking a ball. An average toddler gains 5 to 7 pounds (2.3 to 3.2 kg) each year

A nurse in the mental health clinic is counseling a client with the diagnosis of depression. During the counseling session the client says, "Things always seem the same. They never change." The nurse suspects that the client is feeling hopeless. For what indication of hopelessness should the nurse assess the client? Multiple choice question Outbursts of anger Focused concentration Preoccupation with delusions Intense interpersonal relationships

Clients who are depressed and feeling hopeless also tend to have inappropriate expressions of anger. Depressed clients frequently have a diminished ability to think or concentrate. Preoccupation with delusions is usually associated with clients who have schizophrenia rather than with clients experiencing depression and hopelessness. Clients who are depressed and feeling hopeless tend to be socially withdrawn and do not have the physical or emotional energy for intense interpersonal relationships.

Which statement best explains the focus of a therapeutic milieu management? Multiple choice question Management of a therapeutic milieu is a nursing responsibility. The nurse-patient relationship is dependent upon therapeutic milieu management. Milieu management creates an environment that supports the client's therapeutic care. Creating a therapeutic milieu requires a proactive approach on the part of the nurse.

Milieu management creates an environment that supports the client's therapeutic care. The focus of a therapeutic milieu is the creation and maintenance of an environment that supports and benefits a client toward achieving therapeutic goals. That management of a therapeutic milieu is a nursing responsibility, the nurse-patient relationship is dependent upon therapeutic milieu management, and creating a therapeutic milieu requires a proactive approach on the part of the nurse are true, but these statements do not best explain the focus of the management of the milieu.

During the first meeting of a therapy group, the members become quite uncomfortable. The nurse notes frequent periods of silence, tense laughter, and nervous movement in the group. What does the nurse conclude about these responses? Multiple choice question They require active leader intervention to relieve signs of obvious stress. They indicate unhealthy group processes and an unwillingness to relate openly. They are expected group behaviors because relationships are not yet established. They should be addressed immediately so members will not become too uncomfortable.

They are expected group behaviors because relationships are not yet established. The members have not established trust and are hesitant to discuss problems; the behaviors observed reflect anxiety and insecurity. Requiring active leader intervention to relieve signs of obvious stress can add to the anxiety and insecurity of group members. These behaviors are expected in the early stage of group interaction and are not unhealthy. Immediately addressing them may add to the anxiety and insecurity of the group members.

A client at 37 weeks' gestation is brought to the emergency department because of sudden abdominal pain. Abruptio placentae is suspected, and the client is transferred to the birthing unit. What should the nurse assess the client for? Multiple choice question Bright-red vaginal bleeding and multiple clots Uterine tenderness and increased fetal activity Cessation of contractions and decreased uterine size Concealed hemorrhage and fetal heart rate accelerations

Uterine tenderness and increased fetal activity When the placenta initially separates, the fetus may become hyperactive as a response to acute hypoxia; the uterus is tender because of the accumulation of blood at the abrupted placental site. If bleeding occurs, it is dark red or port wine colored and usually does not clot. The uterus generally enlarges because of an accumulation of blood at the placental site. It is difficult to assess a client for concealed hemorrhage; the fetus must first be assessed for fetal heart tones to determine viability, not for increases or decreases in the heart rate.

The mother of a 30-month-old toddler who has been treated for pinworm infestation is taught how to prevent a recurrence. Which statement by the mother indicates that the teaching has been effective? Multiple selection question "I'll keep the cat off my child's bed." "I'll disinfect my child's room every 2 days." "We'll need to wash all of our sheets every day." "I need to tell the school nurse to have the toilets cleaned." "I'll have the whole family take the medication again in 2 weeks."

Washing clothing and bed linens daily will help limit transmission. Medications such as mebendazole (Vermox), pyrantel pamoate (Antiminth), and pyrvinium (Povan) are effective but must be repeated in 2 weeks to prevent reinfestation. Cats do not transmit pinworms. Disinfection of surfaces does not help prevent transmission. Toilets are not the usual mode of transmission; the rectal-oral cycle must be completed for infestation to occur.

What does "access to care" include according to the Picker Institute's eight dimensions of patient-centered care? Multiple selection question "Clients often need help to complete activities of daily living (ADL)." "Clients expect privacy and to have their cultural values respected." "Clients want to be able to see a specialist when a referral is made." "Clients want to schedule appointments at convenient times without trouble." "Clients need to be able to find conveyance when travelling to different healthcare settings."

"Clients want to be able to see a specialist when a referral is made." "Clients want to schedule appointments at convenient times without trouble." "Clients need to be able to find conveyance when travelling to different healthcare settings." According to the Picker Institute's eight dimensions of patient-centered care, " access to care" includes several features. Clients want to be able to see a professional when a referral is made. Clients want to schedule appointments at convenient times without trouble. Clients need to be able to find conveyance when travelling to different healthcare settings. According to the Picker Institute's eight dimensions of patient-centered care, "physical comfort" includes aspects such as clients requiring help to complete activities of daily living (ADL) and clients expecting privacy and respect towards their cultural values.

When a 12-year-old boy who sustained several tick bites on a camping trip becomes ill, he is told that he may have Lyme disease. He asks the nurse, "What is Lyme disease?" What is the best response by the nurse? Multiple choice question "I can see that you're concerned. Tell me what you want to know." "The infection is caused by a spirochete. It can be cured with penicillin." "The tick bites gave you an infection. There is medication that will treat it." "You sound upset. Don't worry—we have medicine that will make you better."

"The tick bites gave you an infection. There is medication that will treat it." Telling the child that a tick bite caused the disease and that it is curable is a straightforward, truthful answer at a level that a 12-year-old child will comprehend. Just identifying the child's feelings disregards the fact that the child has asked a question that requires an answer. The child may not understand scientific terminology. Telling the child not to worry is demeaning and avoids answering the question.

An infant who was in a motor vehicle collision has undergone open repair of a fractured sternum and now has a chest tube. What should the nurse explain to the infant's parents about the chest tube? Multiple choice question "The tube doesn't cause discomfort. It's been put in place for emergency use." "The tube will be taken out once your baby is stable and oral feedings are started." "The tube has been placed to drain the air that entered the chest cavity during surgery." "The tube drains the air that accumulated in your baby's chest after the lung was punctured."

"The tube has been placed to drain the air that entered the chest cavity during surgery." The chest was opened during surgery for sternal repair, allowing air to enter the pleural space; the air must be removed for the lungs to expand. Chest tubes are uncomfortable; also, saying it is in place for emergency use discounts the importance of the chest tube to the infant's respiratory status. The chest tube is unrelated to the infant's ability to ingest oral feedings. The data do not indicate the presence of a punctured lung.

A nurse is caring for a client with a diagnosis of conversion disorder manifesting as paralysis of the legs. Which is the most therapeutic nursing intervention? Multiple choice question Encouraging the client to try to walk Explaining to the client that there is nothing wrong Avoiding focusing on the client's physical symptoms Helping the client follow through with the physical therapy plan

Avoiding focusing on the client's physical symptoms The physical symptoms are not the client's major problem and therefore should not be the focus of care. This is a psychological problem, and the focus should be in this domain. Encouraging the client to try to walk is focusing on the physical symptom of the conflict; the client is not ready to give up the symptom. The disorder operates on an unconscious level but is very real to the client; saying there is nothing wrong denies feelings. Psychotherapy, not physical therapy, is needed at this time.

A block nurse is caring for an elderly couple in the neighborhood. What kind of service does block nursing offer to the elderly clients? Multiple choice question Diagnostics Health screening Running errands Communicable disease control

Block nursing involves the services of nurses living within a neighborhood. The nurse generally provides services for older clients or those who are unable to leave their homes. Therefore running errands is one of the services offered by the block nurse. The primary health care provider's offices provide primary health care, which includes diagnostics and treatment. School health, occupational health, primary health care provider's offices, and community health centers provide health screening. Communicable disease control is offered by occupational health services.

A nurse is caring for a client with the diagnosis of schizophrenia. During assessment the nurse identifies both positive (type I) and negative (type II) signs and symptoms. Which clinical findings should the nurse document as positive? Multiple selection question Anergy Flat affect Social withdrawal Disorganized thoughts Auditory hallucinations

Disorganized thoughts (e.g., derailment, tangentiality, illogicality, incoherence, and circumstantiality) are a positive sign of schizophrenia. Positive signs and symptoms, referred to as "florid psychotic symptoms," are related to alterations in thinking, speech, perception, and behavior. They usually respond to antipsychotic medications. Positive symptoms reflect an excess or distortion of function and include delusions, hallucinations, increased speech production with associations, and bizarre behavior. A lack of energy (anergy) is a negative symptom associated with schizophrenia. Negative symptoms reflect a lessening or loss of normal function. A lack of emotional expression (flat affect) is a negative sign associated with schizophrenia. Inadequate social skills leading to withdrawal and isolation are negative symptoms associated with schizophrenia.

A 10-year-old child undergoes open heart surgery to repair a cardiac defect. The healthcare provider informs the parents that antibiotics are required before any dental work is performed. Later the parents ask the nurse why this is necessary. When responding, the nurse explains that this is done to prevent what type of infection? Multiple choice question Gingivitis Pericarditis Myocarditis Endocarditis

Endocarditis The administration of antibiotics before an invasive procedure can prevent subacute bacterial endocarditis, which may occur in children and adults with heart abnormalities. The endocardium is the lining membrane of the cavities of the heart and the connective tissue bed on which it lies. Gingivitis, an inflammation of the gums, frequently is related to the accumulation of food particles in the crevices between the gums and teeth. Antibiotics are not given to prevent this condition, although they may be prescribed in the presence of an infection of the gums. The endocardium is more susceptible to infection than is the pericardium. The pericardium is the fibroserous sac enclosing the heart. The myocardium is the middle layer of the heart wall, composed of cardiac muscle; myocarditis is an infection of the myocardium, which is an unlikely sequela of dental procedures.

The nurse notes that a 4-year-old child is having difficulty relating to some of the children in the playroom. What does the nurse identify as the reason that this problem is not unexpected? Multiple choice question At this age preschoolers engage only in parallel play. At this age preschoolers are extremely dependent on their parents. Fierce temper tantrums and negativism are typical behaviors of preschoolers. Exaggerating and boasting to impress others are typical behaviors of preschoolers.

Exaggerating and boasting to impress others are typical behaviors of preschoolers. It is common for 4-year-old children to boast and exaggerate and to be impatient, noisy, and selfish. More advanced, cooperative play is expected of 4-year-old children. Extreme dependence on parents is unusual in 4-year-old children because they are striving for more initiative and less dependence. The toddler's tendency toward tantrums and negativism should have waned by 4 years of age.

A nurse is caring for a client with bipolar disorder, depressive episode. What should the nurse's initial objective for this client be? Multiple choice question Feeling comfortable with the nurse Investigating new leisure activities Participating in small group activities Initiating conversations about feelings

Feeling comfortable with the nurse Before therapy can begin, a trusting relationship must be developed. A client with major depression will not have the impetus or energy to investigate new leisure activities. Participating in small group activities is not appropriate initially; the client does not have the physical or emotional energy to interact with a small group of people. Initiating conversations about feelings will not be successful unless the client develops a trusting, comfortable relationship with the nurse.

A primigravida client gave birth by vaginal delivery 24 hours ago. Which findings would be considered normal? Multiple choice question Fundus firm at the umbilicus; moderate lochia rubra; voiding quantity sufficient; colostrum present Fundus firm, one fingerbreadth above the umbilicus; scant lochia alba; voided twice, 500 mL, 400 mL; breasts heavy Fundus firm, two fingerbreadths above the umbilicus; moderate lochia serosa; voided once, 200 mL; colostrum present Fundus firm, two fingerbreadths below the umbilicus; moderate serosa alba; voiding quantity sufficient; breasts engorged

Fundus firm at the umbilicus; moderate lochia rubra; voiding quantity sufficient; colostrum present Twenty-four hours after delivery, the fundus is usually at the umbilicus and moderate lochia rubra is expected. Colostrum is present, and the breast milk usually comes in on day 3 after delivery. A fundus two fingerbreadths above the umbilicus may indicate a full bladder, and lochia serosa occurs during days 4 through 10. Voiding just 200 mL since delivery is inadequate. The presence of colostrum is normal. A fundus that is firm at two fingerbreadths under the umbilicus is acceptable, but lochia alba occurs after the 10th postpartum day. The milk would have had to come in for the breasts to be engorged, which does not typically occur until day 3. Scant lochia alba would not occur until day 10; nor would the milk supply be established.

A young adolescent is found to have anorexia nervosa. What does the nurse understand probably precipitated the anorexia nervosa? Multiple choice question The acting out of aggressive impulses, resulting in feelings of hopelessness An unconscious wish to punish a parent who tries to dominate the adolescent's life The inability to deal with being the center of attention in the family and a desire for independence An inaccurate perception of hunger stimuli and a struggle between dependence and independence

Inaccurate perception of hunger stimuli and a struggle between dependence and independence are theoretical explanations for the development of anorexia nervosa[1][2] . Acting-out and the wish to punish a domineering parent do not play a role in the development of anorexia nervosa. The inability to be the center the family's attention has not been correlated with anorexia nervosa.

A nurse is caring for a group of postpartum clients. Which client is at the highest risk for disseminated intravascular coagulation (DIC)? Multiple choice question Gravida III with twins Gravida V with endometriosis Gravida II who had a 9-lb baby Gravida I who has had an intrauterine fetal death

Intrauterine fetal death is one of the risk factors for DIC; other risk factors include abruptio placentae, amniotic fluid embolism, sepsis, and liver disease. Multiple pregnancy, endometriosis, and increased birthweight are not risk factors for DIC.

The personality characteristics of a client with an antisocial personality disorder make it difficult for family members to interact and maintain a healthy relationship. What are common characteristics of an antisocial personality? Multiple selection question Aloof Suspicious Perfectionist Irresponsible Manipulative

Irresponsible Manipulative People with antisocial personalities are often irresponsible, amoral, and dishonest and do not learn from negative experiences. People with antisocial personalities are often charming and calculating when exploiting others; they show no remorse for hurting others and do not develop insight into predictable consequences. Aloofness is associated with the schizoid personality. Suspiciousness is associated with the paranoid personality. Perfectionism is associated with the paranoid personality.

Which nursing action best promotes parent-infant attachment behaviors? Multiple choice question Restricting visitors on the postpartum unit Supporting rooming-in with parent-infant care Encouraging the mother to choose breast-feeding Keeping the new family together immediately after the birth

Keeping the new family together immediately after the birth Research strongly supports the theory that there is a sensitive period during the first few hours of life that is important for the promotion of parent-infant attachment. Parent-infant bonding can take place with or without visitors. Encouraging rooming-in is helpful because it increases the amount of contact between the parents and the newborn; however, it is not as significant as those critical first few hours after the birth. Contact with the newborn can be achieved with breast-feeding or formula feeding; it is the contact, not the method, that promotes bonding.

A client who is to be discharged from an inpatient mental health facility is referred to a mental health daycare center in the community. What should the nurse identify as the primary reason for this referral? Multiple choice question Improving social skills Getting out of the house for a few hours daily Maintaining gains achieved during hospitalization Avoiding direct confrontation with the community

Maintaining gains achieved during hospitalization The daycare center provides the client with a therapeutic setting for a few hours each day during the transitional stage between hospital and total discharge. The goal is to maintain and enhance progress made during inpatient treatment. Daycare treatment may improve social skills or allow the client to get out of the house for a few hours, but neither is its primary purpose. Avoiding direct confrontation with the community may help during the transition stage, but it is not the primary goal of daycare.

A nurse who is assessing a full-term newborn elicits the magnet reflex. How is this reflex elicited? Multiple choice question Striking the surface of the newborn's crib suddenly Stroking the outer sole of the newborn's foot from the heel to the little toe Maintaining the supine position and applying pressure to the soles of the newborn's feet Holding the newborn's body upright and allowing the feet to touch the surface of the crib

Maintaining the supine position and applying pressure to the soles of the newborn's feet Applying pressure to the sole of the foot produces the magnet reflex, in which the legs extend in response to the pressure on the soles of the feet. Jarring the crib produces a startle response (Moro reflex). Stroking the outer sole of the foot from the heel to the little toe produces the Babinski or plantar reflex; all of the toes hyperextend. Allowing the feet to touch the surface of the crib produces the stepping reflex, in which one foot is placed before the other in a simulated walk, with the weight on the toes.

A client with the diagnosis of schizophrenia, paranoid type, appears very suspicious of the nurse. What is the most effective therapeutic nursing approach? Multiple choice question Assigning various caregivers to the client Making brief, frequent contacts with the client Initiating a discussion about the client's thoughts Allowing the client to stay alone without interruption

Making brief, frequent contacts with the client Brief, frequent contacts are less threatening and help build trust. Assigning various caregivers to the client will increase suspiciousness; the client needs consistent caregivers to help increase the level of trust. Initiating a discussion about the client's thoughts supports the client's delusions and thus increases suspiciousness. Allowing the client to stay alone without interruption does not engage the client in a therapeutic manner and reinforces the social withdrawal common with paranoia.

A client at 10 weeks' gestation calls the clinic and tells a nurse that she has morning sickness and cannot control it. What should the nurse suggest to promote relief? Multiple choice question "Eat dry crackers before you get out of bed." "Increase your fat intake before bedtime." "Drink high-carbohydrate fluids with meals." "Eat two small meals a day and a snack at noon."

Nausea and vomiting in the morning occur in almost 50% of all pregnancies. Eating dry crackers before getting out of bed in the morning is a simple remedy that may provide relief. Increasing fat intake does not relieve the nausea. Drinking high-carbohydrate fluids with meals is not helpful; separating fluids from solids at mealtime is more advisable. Eating two small meals a day and a snack at noon does not meet the nutritional needs of a pregnant woman, nor will it relieve nausea. Some women find that eating five or six small meals daily instead of three large ones is helpful.

When is the most appropriate time for the nurse to plan for chest percussion and postural drainage for a toddler with cystic fibrosis? Multiple choice question After suctioning Before aerosol therapy One hour before meals Fifteen minutes after meals

One hour before meals Performing chest percussion and postural drainage 1 hour before meals will give the child an opportunity to rest before eating. The child should be encouraged to cough; if this is not effective, suctioning may be done after chest percussion and postural drainage. Chest percussion and drainage should be done after aerosol therapy. Performing chest percussion and postural drainage 15 minutes after a meal may cause the child to vomit.

An 8-year-old child is found to have oppositional defiant disorder. What behavior noted by the nurse supports this diagnosis? Multiple choice question Easily distracted Argues with adults Lies to obtain favors Initiates physical fights!

Oppositional defiant disorder is a repeated pattern of negativistic, disobedient, hostile, defiant behavior toward authority figures, usually exhibited before 8 years of age. Easy distraction, associated with attention deficit-hyperactivity disorder, reflects an inability to sustain focus on a task. Lying to obtain favors is associated with conduct disorder and reflects a violation of a societal norm. Initiating physical fights and violating the rights of others are associated with conduct disorder.

A 6-year-old child who just completed a cycle of chemotherapy is to be discharged home. What is the priority teaching point at discharge? Multiple choice question Purchasing a wig Ensuring adequate rest Offering sufficient fluids Performing thorough hand washing

Performing thorough hand washing Effective hand washing helps prevent infection, and this is the most important intervention in the care of a child with a suppressed immune system. Although purchasing a wig may be important for the child's self-image, it is not the priority. Although providing rest and offering sufficient fluids are also important, neither is the priority.

Why would a client with acquired immunodeficiency syndrome (AIDS) be administered pregabalin? Multiple choice question To reduce neuropathic pain To reduce cognitive difficulty To reduce swallowing difficulty To reduce muscle and joint pain

Pregabalin is indicated for neuropathic pain based on its mechanism of interference with nerve signaling. Clients with AIDS generally exhibit emotional and behavioral changes, which can be managed with appropriate antidepressants and anxiolytics. AIDS clients who experience difficulty swallowing may have candidal esophagitis; this condition can be managed with antifungal mediations such as fluconazole or amphotericin B. Traditional analgesics are used to manage joint and muscle pain.

While assessing a four-year-old child, a nurse learns that the child is able to judge an action as good because it resulted in a reward. Which stage of moral development does the nurse note down in the child's health record? Multiple choice question Phallic stage Oedipal stage Instrumental orientation Punishment and obedience orientation

Punishment and obedience orientation At the age of two to four years, children judge whether an action is good or bad according to whether it results in reward or punishment. This stage of moral development in the child is known as punishment and obedience orientation. At the age of seven years, children direct their actions toward their needs and are said to be in the instrumental orientation of moral development. Freud described phallic and Oedipal stages as phases of psychologic development in boy and girl children.

The nurse observes that a child fails to make eye contact and has poor impulse control. Upon further assessment, the nurse finds that the parent is an alcoholic and often neglects the child. What can be said about the child? Multiple choice question The child needs to be screened for autism. The child is experiencing separation anxiety. The child feels solitary because of the parent's behavior. The child has developed reactive attachment disorder (RAD)

RAD is a psychological and developmental disorder that occurs in children who are neglected by their primary caregivers. Children with RAD are not cuddly with parents and fail to make eye contact. They also exhibit poor impulse control and may be destructive to themselves and others. Poor eye contact is seen in autistic children as well, but in this case, there is parental neglect that indicates RAD. Separation anxiety is indicated by crying and screaming when the parent leaves the child. Feelings of solitariness do not result in poor impulse control or eye contact.

A nurse is assessing an older adult with the diagnosis of dementia. Which manifestations are expected in this client? Multiple selection question Resistance to change Inability to recognize familiar objects Preoccupation with personal appearance Inability to concentrate on new activities or interests Tendency to dwell on the past and ignore the present

Resistance to change is a clinical finding associated with dementia; these clients need structure and routines[1][2][3]. An inability to recognize familiar objects (agnosia) is a typical cognitive dysfunction associated with dementia. A short attention span and little or no interest in new activities are typical of dementia. The past, rather than the threatening present, is where these clients feel comfortable. Clients with delirium, dementia, and other cognitive disorders rarely express any concern about personal appearance. The staff must meet most of these clients' personal needs.

When a preterm newborn requires oxygen, the nurse in the neonatal intensive care unit monitors and adjusts the oxygen concentration. Which complication do these adjustments attempt to prevent? Multiple choice question Cataracts Strabismus Ophthalmia neonatorum Retinopathy of prematurity

Retinopathy of prematurity Retinopathy of prematurity is caused by the high concentration of oxygen that may have to be used to support some preterm neonates; oxygen must be administered cautiously and, depending on the neonate's blood oxygen level, adjusted accordingly. Cataracts and strabismus (crossed eyes) are not caused by a high oxygen concentration. Ophthalmia neonatorum refers to an inflammation of the eyes caused by a gonorrheal or chlamydial infection contracted as the fetus passes through the birth canal.

The most appropriate method for a nurse to evaluate the effects of the maternal blood glucose level in the infant of a diabetic mother is by performing a heel stick blood test on the newborn. What specifically does this test determine? Multiple choice question Blood acidity Glucose tolerance Serum glucose level Glycosylated hemoglobin level

Serum glucose level

A nurse is caring for a newly admitted client with obsessive-compulsive disorder. When should the nurse anticipate that the client's anxiety level will increase? Multiple choice question As the day progresses When family members visit During a physical assessment by the nurse When limits are set on the performance of a ritual

Setting limits on the performance of a ritual will increase the client's anxiety. The ritual is a defense that the client needs at this time to control anxiety. The client needs time to develop other defenses before the ritual can be limited. The precipitation of anxiety in a client with obsessive-compulsive disorder is usually unrelated to the time of day. Visits from family members may or may not precipitate anxiety. Researchers have implicated trauma to the basal ganglia or cortical connections or a genetic predisposition as the origin of obsessive-compulsive disorder. A physical assessment by the nurse may or may not precipitate anxiety. The presentation of a nonjudgmental, supportive attitude by the nurse should decrease, not increase, anxiety.

A nurse believes that a client who is being discharged after a physical attack by an unknown assailant will benefit from further care to help resolve residual feelings. For what type of therapy should the nurse refer the client? Multiple choice question Psychotherapy that emphasizes desensitization Short-term therapy emphasizing crisis intervention Long-term therapy with a psychoanalytical emphasis Group therapy with a behavior modification component

Short-term therapy emphasizing crisis intervention Crisis intervention helps the client put the event in perspective and resolve feelings so the individual can resume life within a short time. Desensitization is effective if the victim develops phobias as a result of the physical attack. Unless there are complicating factors, long-term therapy is not indicated. The client's behavior did not precipitate the attack.

The nurse finds a disturbed, acting-out client in the fetal position. What is the most appropriate intervention for the nurse? Multiple choice question Sitting down beside the client and saying, "I'm here to spend time with you." Going to the client and saying, "I'll be waiting for you by the chairs, so please get up and join me." Tapping the client gently on the shoulder to get the client's attention and then staying with the client Leaving the client alone because the behavior demonstrates that the client is too regressed to benefit from verbal communication

Sitting down beside the client and saying, "I'm here to spend time with you." Sitting down beside the client and saying, "I'm here to spend time with you" shows acceptance of the client at the current level and allows the client to set the pace of the relationship. Tapping the client gently on the shoulder to get the client's attention and then staying with the client could be misinterpreted by any client and could precipitate an aggressive response. Going to the client and saying, "I'll be waiting for you by the chairs, so please get up and join me" asks the client to reach out to the nurse; in the therapeutic relationship the nurse must reach out to the client. Even if the client is too withdrawn to respond, the nurse's physical presence can be reassuring, so leaving the client alone is not therapeutic

A nurse is conducting the Mini-Mental Status examination on an older client. What should the nurse ask the client to do when testing short-term memory? Multiple choice question Subtract serial sevens from 100. Copy one simple geometric figure. State three random words mentioned earlier in the exam. Name two common objects when the nurse points to them

State three random words mentioned earlier in the exam. Stating three random words mentioned earlier in the examination is a test of the client's ability to recall from short-term memory. Subtracting serial sevens from 100 is a test of the ability to calculate and pay attention. Copying one simple geometric figure is a test of visual comprehension. Naming two common objects when the nurse points to them is a test of verbal skills to identify aphasia.

What is a priority nursing intervention in the care of a drug-dependent mother and infant? Multiple choice question Supporting the mother's positive responses toward her infant Requesting that family members share responsibility for infant care Keeping the infant separated from the mother until the mother is drug free Helping the mother understand that the infant's problems are a result of her drug intake .

Supporting the mother's positive responses toward her infant A nurse should attempt to support the mother-child relationship; the mother is experiencing a developmental crisis while coping with drug addiction and possibly guilt. It is the client's right to decide who will share in the care of her child. The client needs contact with her new infant to facilitate bonding. Helping the mother understand that the infant's problems are a result of her drug intake will make the client feel guilty and will not facilitate positive action at this point

Phototherapy is prescribed for a preterm neonate with hyperbilirubinemia. Which nursing intervention is appropriate to reduce the potentially harmful side effects of the phototherapy? Multiple choice question Covering the trunk to prevent hypothermia Using shields on the eyes to protect them from the light Massaging vitamin E oil into the skin to minimize drying Turning after each feeding to reduce exposure of each surface area

Using shields on the eyes to protect them from the light The lights used for phototherapy can damage the infant's eyes, and eye shields are standard equipment. Maximal effectiveness is achieved when the infant's entire skin surface is exposed to the light. Vitamin E oil massage is contraindicated, because it can cause burns and result in an overdose of the vitamin. The infant should be turned every 2 hours regardless of feeding times so that all body surfaces are exposed to the light and no single body surface is overexposed.

All women of childbearing age are advised to include at least 400 mcg of folic acid in the daily diet to decrease the risk of neural tube defects in pregnancy. What should the nurse recommend to meet the recommendation? Multiple selection question Vitamin A Vitamin B 6 Vitamin B 9 Vitamin B 12 Legumes, dark-green leafy vegetables, and citrus fruits Eggs, meat, and poultry

Vitamin B 9 Legumes, dark-green leafy vegetables, and citrus fruits Vitamin B 9 is folic acid, and legumes, dark-green leafy vegetables, and citrus fruits are natural sources of folic acid. Most women receive adequate vitamin A in their diets, and too much may cause birth defects. Vitamin B 6 aids in metabolism conversion and the formation of red blood cells. Vitamin B 12 is associated with nerve cells and red blood cells. Eggs, meat, and poultry are sources of vitamin B 12.


संबंधित स्टडी सेट्स

CPR/First-Aid/Choking Victim Study Guide

View Set